Menu Close

lim-n-k-1-n-n-2-k-




Question Number 151767 by mathdanisur last updated on 22/Aug/21
lim_(n→∞)  Σ_(k=1) ^∞  (n/(n^2  + k)) = ?
$$\underset{\boldsymbol{\mathrm{n}}\rightarrow\infty} {\mathrm{lim}}\:\underset{\boldsymbol{\mathrm{k}}=\mathrm{1}} {\overset{\infty} {\sum}}\:\frac{\mathrm{n}}{\mathrm{n}^{\mathrm{2}} \:+\:\mathrm{k}}\:=\:? \\ $$
Answered by Olaf_Thorendsen last updated on 22/Aug/21
my calculous was false.  sorry.
$$\mathrm{my}\:\mathrm{calculous}\:\mathrm{was}\:\mathrm{false}. \\ $$$$\mathrm{sorry}. \\ $$
Commented by puissant last updated on 22/Aug/21
monsieur on ne peut pas utiliser  RIEMANN car le ′′k′′ du denominateur  n′est pas au carre^� ..  en fait c′est ce que je pense..
$${monsieur}\:{on}\:{ne}\:{peut}\:{pas}\:{utiliser} \\ $$$${RIEMANN}\:{car}\:{le}\:''{k}''\:{du}\:{denominateur} \\ $$$${n}'{est}\:{pas}\:{au}\:{carr}\acute {{e}}.. \\ $$$${en}\:{fait}\:{c}'{est}\:{ce}\:{que}\:{je}\:{pense}.. \\ $$
Commented by Olaf_Thorendsen last updated on 22/Aug/21
Exact ! Bien vu !
$$\mathrm{Exact}\:!\:\mathrm{Bien}\:\mathrm{vu}\:! \\ $$
Answered by mindispower last updated on 22/Aug/21
Σ_(k=1) ^n (1/(n(1+(k/n^2 ))))=S  1−x≤(1/(1+x))≤1,∀x≥0  (1/n)−(k/n^3 )≤(1/(n(1+(k/n^2 ))))≤(1/n)  Σ_(k=1) ^n ((1/n)−(k/n^3 ))≤Σ_(k=1) ^n (1/(n(1+(k/n^2 ))))≤Σ_(k=1) ^n (1/n)  1−(1/n^3 ).(((n(n+1))/2))≤S≤1⇒S→1
$$\underset{{k}=\mathrm{1}} {\overset{{n}} {\sum}}\frac{\mathrm{1}}{{n}\left(\mathrm{1}+\frac{{k}}{{n}^{\mathrm{2}} }\right)}={S} \\ $$$$\mathrm{1}−{x}\leqslant\frac{\mathrm{1}}{\mathrm{1}+{x}}\leqslant\mathrm{1},\forall{x}\geqslant\mathrm{0} \\ $$$$\frac{\mathrm{1}}{{n}}−\frac{{k}}{{n}^{\mathrm{3}} }\leqslant\frac{\mathrm{1}}{{n}\left(\mathrm{1}+\frac{{k}}{{n}^{\mathrm{2}} }\right)}\leqslant\frac{\mathrm{1}}{{n}} \\ $$$$\underset{{k}=\mathrm{1}} {\overset{{n}} {\sum}}\left(\frac{\mathrm{1}}{{n}}−\frac{{k}}{{n}^{\mathrm{3}} }\right)\leqslant\underset{{k}=\mathrm{1}} {\overset{{n}} {\sum}}\frac{\mathrm{1}}{{n}\left(\mathrm{1}+\frac{{k}}{{n}^{\mathrm{2}} }\right)}\leqslant\underset{{k}=\mathrm{1}} {\overset{{n}} {\sum}}\frac{\mathrm{1}}{{n}} \\ $$$$\mathrm{1}−\frac{\mathrm{1}}{{n}^{\mathrm{3}} }.\left(\frac{{n}\left({n}+\mathrm{1}\right)}{\mathrm{2}}\right)\leqslant{S}\leqslant\mathrm{1}\Rightarrow{S}\rightarrow\mathrm{1} \\ $$$$ \\ $$$$ \\ $$
Answered by Kamel last updated on 22/Aug/21
Ω=lim_(n→∞)  Σ_(k=1) ^n  (n/(n^2  + k))      =lim_(n→+∞) n(Ψ(n^2 +n+1)−Ψ(1+n^2 ))     =lim_(n→+∞) n(Ln(n^2 +n+1)−Ln(n^2 ))     =lim_(t→0^+ ) ((Ln(1+t+t^2 ))/t)=1
$$\Omega=\underset{\boldsymbol{\mathrm{n}}\rightarrow\infty} {\mathrm{lim}}\:\underset{\boldsymbol{\mathrm{k}}=\mathrm{1}} {\overset{{n}} {\sum}}\:\frac{\mathrm{n}}{\mathrm{n}^{\mathrm{2}} \:+\:\mathrm{k}}\: \\ $$$$\:\:\:=\underset{{n}\rightarrow+\infty} {{lim}n}\left(\Psi\left({n}^{\mathrm{2}} +{n}+\mathrm{1}\right)−\Psi\left(\mathrm{1}+{n}^{\mathrm{2}} \right)\right) \\ $$$$\:\:\:=\underset{{n}\rightarrow+\infty} {{lim}n}\left({Ln}\left({n}^{\mathrm{2}} +{n}+\mathrm{1}\right)−{Ln}\left({n}^{\mathrm{2}} \right)\right) \\ $$$$\:\:\:=\underset{{t}\rightarrow\mathrm{0}^{+} } {{lim}}\frac{{Ln}\left(\mathrm{1}+{t}+{t}^{\mathrm{2}} \right)}{{t}}=\mathrm{1} \\ $$

Leave a Reply

Your email address will not be published. Required fields are marked *